Đến nội dung

binhnhaukhong nội dung

Có 343 mục bởi binhnhaukhong (Tìm giới hạn từ 17-04-2020)



Sắp theo                Sắp xếp  

#714754 $\dfrac{1}{a^2+bc}+\dfrac{1}...

Đã gửi bởi binhnhaukhong on 24-08-2018 - 17:20 trong Bất đẳng thức - Cực trị

Bài 10: Cho 3 số thực không âm $a, b, c$ sao cho không có 2 số nào đồng thời bằng 0. Chứng minh bất đẳng thức:
$$\dfrac{1}{a^2+bc}+\dfrac{1}{b^2+ac}+\dfrac{1}{c^2+ab} \geqslant \dfrac{3(a+b+c)^2}{2(a^2+b^2+c^2)(ab+bc+ca)}$$
$$\text{- Phạm Hữu Đức -}$$

Nhân cả 2 vế BĐT cần chứng minh với $ab+bc+ac$:

 

$$\frac{a(b+c)+bc}{a^2+bc}+\frac{b(a+c)+ac}{b^2+ac}+\frac{c(a+b)+ab}{c^2+ab}\geq \frac{3(a+b+c)^2}{2(a^2+b^2+c^2)}$$

 

BĐT trên là tổng của 2 BĐT sau:

 

$$\frac{a(b+c)}{a^2+bc}+\frac{b(a+c)}{b^2+ac}+\frac{c(a+b)}{c^2+ab}\geq \frac{(a+b+c)^2}{a^2+b^2+c^2} (1)$$

 

 

$$\frac{bc}{a^2+bc}+\frac{ac}{b^2+ac}+\frac{ab}{c^2+ab}\geq \frac{(a+b+c)^2}{2(a^2+b^2+c^2)} (2)$$

 

(1) có thể chứng minh bằng Vornicu Schur, (2) có thế dùng Cauchy- Schwarz để chứng minh.

 

Nếu nhân cả 2 vế với $(a^2+b^2+c^2)$:

 

$$\frac{b^2+c^2-bc}{a^2+bc}+\frac{c^2+a^2-ac}{b^2+ac}+\frac{a^2+b^2-ab}{c^2+ab}+3\geq \frac{3(a+b+c)^2}{2(ab+bc+ac)}$$

 

Mà $\sum \frac{b^2+c^2-bc}{a^2+bc}\geq \frac{(2a^2+2b^2+2c^2-ab-bc-ac)^2}{\sum (a^2+bc)(b^2+c^2-bc)}$

 

Cũng cho kết quả đúng.




#711476 $\sum \sqrt{a^2+4bc}\geq \sqrt{a^2+b^...

Đã gửi bởi binhnhaukhong on 23-06-2018 - 23:17 trong Bất đẳng thức - Cực trị

Cho 3 số thực không âm $a, b, c$. Chứng minh bất đẳng thức:

$$\sqrt{a^2+4bc}+\sqrt{b^2+4ac}+\sqrt{c^2+4ab} \geq \sqrt{a^2+b^2+c^2+14(ab+bc+ca)}$$

Nguồn: Không rõ

Giả sử $c= min${$a,b,c$}, ta có bổ đề sau:

 

$$\sqrt{(a^2+4bc)(b^2+4ac)}\geq ab+2c(a+b)$$

 

và $\sqrt{a^2+4bc}+\sqrt{b^2+4ac}\geq \sqrt{(a+b)^2+8c(a+b)}$

 

Tương tự ta có bài toán sau:

 

Cho các số thực $a,b,c$ không âm. Chứng minh rằng ta có BĐT sau:

 

$$\sqrt{a^2+9bc}+\sqrt{b^2+9ac}+\sqrt{c^2+9ab}\geq \sqrt{a^2+b^2+c^2+23(ab+bc+ac)}$$




#673545 Đề Olympic Toán SV ĐHBK Hà Nội

Đã gửi bởi binhnhaukhong on 05-03-2017 - 20:10 trong Thảo luận về các kì thi, các kì kiểm tra Toán sinh viên

File ảnh mình đính kèm bên dưới :)

Hình gửi kèm

  • 16938497_1842886562620168_2661335316910832802_n.jpg



#614303 Tiếp sức bất đẳng thức

Đã gửi bởi binhnhaukhong on 11-02-2016 - 23:03 trong Bất đẳng thức và cực trị

 

Bài 44:cho a,b,c>0 tim min $\frac{b(a-c)}{c(a+b)}+\frac{c(3b+a)}{a(b+c)}+\frac{2c(a-b)}{b(a+c)}$

 

 

Đề này mới chuẩn chứ!




#614031 $\sum \frac{1}{a^{2}+ab+b^{2...

Đã gửi bởi binhnhaukhong on 10-02-2016 - 20:49 trong Bất đẳng thức - Cực trị

Bất đẳng thức này sai với $a=1,\,b=2,\,c=1.$

Anh ơi em gõ nhầm đấy anh ạ, chả ai nhìn ra :3.Giờ thì ổn rồi ạ.




#614030 Tiếp sức bất đẳng thức

Đã gửi bởi binhnhaukhong on 10-02-2016 - 20:45 trong Bất đẳng thức và cực trị

Chú ý rằng $\frac{ab+bc+ca}{(a+b)^2} = \frac{c}{a+b}+\frac{ab}{(a+b)^2},$ nên bất đẳng thức trên tương đương với

\[\frac{a}{b+c}+\frac{b}{c+a}+\frac{c}{a+b}+\frac{ab}{(a+b)^2}+\frac{bc}{(b+c)^2}+\frac{ca}{(c+a)^2} \leqslant \frac{a^2+b^2+c^2}{ab+bc+ca}+\frac{5}{4}.\]

Mặt khác ta lại có

\[\frac{ab}{(a+b)^2}+\frac{bc}{(b+c)^2}+\frac{ca}{(c+a)^2} \leqslant \frac{1}{4}+\frac{4abc}{(a+b)(b+c)(c+a)},\]

nên ta chỉ cần chứng minh

\[\frac{a}{b+c}+\frac{b}{c+a}+\frac{c}{a+b}+\frac{4abc}{(a+b)(b+c)(c+a)} \leqslant \frac{a^2+b^2+c^2}{ab+bc+ca}+1,\]

Đặt $p=a+b+c,\,q=ab+bc+ca$ và $r=abc$ thì bất đẳng thức trên trở thành

\[pq^2 \geqslant (p^2+6q)r.\]

Bất đẳng thức này đúng vì $r \leqslant \frac{q^2}{3p}$ và

\[pq^2 - (p^2+6q) \cdot \frac{q^2}{3p} = \frac{2q^2(p^2-3q)}{3p} \geqslant 0.\]

Bài toán được chứng minh.

Có lẽ chưa cần dùng đến $p,q,r$ đâu anh :)

 

$$4\left(c+\frac{ab}{a+b}\right)^2 \leq 4c^2+2c(a+b)+ab$$




#613951 Tiếp sức bất đẳng thức

Đã gửi bởi binhnhaukhong on 10-02-2016 - 14:02 trong Bất đẳng thức và cực trị

 

Bài 61: Cho $a,b,c$ không âm thỏa $ab+bc+ac>0$.Chứng minh rằng:

 

$$\frac{1}{(a+b)^2}+\frac{1}{(b+c)^2}+\frac{1}{(c+a)^2}\leq \frac{4(a^2+b^2+c^2)+5(ab+bc+ac)}{4(ab+bc+ac)^2}$$




#613901 $\sum \frac{1}{a^{2}+ab+b^{2...

Đã gửi bởi binhnhaukhong on 10-02-2016 - 10:59 trong Bất đẳng thức - Cực trị

Cho a,b,c>0.CMR

     

$\sum \frac{1}{a^{2}+ab+b^{2}}\geq \frac{21}{2(a^{2}+b^{2}+c^{2})+5(ab+bc+ca)}$

  p/s: Mong ae có 1 lời giải đẹp cho bài toán

Có 2 cách cho bài này, 1 là C-S, thứ 2 là dồn biến, không đẹp lắm:

 

$$\sum \frac{1}{b^2+bc+c^2}=\sum \frac{(5a+b+c)^2}{(5a+b+c)^2(b^2+bc+c^2)}\geq \frac{49(a+b+c)^2}{\sum (5a+b+c)^2(b^2+bc+c^2)}$$

 

Bây giờ phải chỉ ra: 

 

$$\frac{49(a+b+c)^2}{\sum (5a+b+c)^2(b^2+bc+c^2)}\geq \frac{21}{2(a^2+b^2+c^2)+5(ab+bc+ac)}$$

 

BĐT này đúng.Dùng Schur để chứng minh.




#603140 $(x+y)(y+z)(z+x) \geq \frac{8}{3}(x+y+z)...

Đã gửi bởi binhnhaukhong on 14-12-2015 - 16:29 trong Bất đẳng thức và cực trị

Cho x,y,z > 0 thỏa mãn xyz=1. Chứng minh rằng :
$(x+y)(y+z)(z+x) \geq \frac{8}{3}(x+y+z)$

Chứng minh lại bổ đề sau( mất vài dòng):

 

$$(x+y)(y+z)(x+z) \geq \frac{8}{9}(x+y+z)(xy+yz+xz)$$




#603139 Chứng minh rằng: $\sum \dfrac{a^4}{b^2}...

Đã gửi bởi binhnhaukhong on 14-12-2015 - 16:27 trong Bất đẳng thức - Cực trị

Bài toán. Cho các số thực dương $a,b,c$ thỏa mãn $abc=1$. Chứng minh rằng:

$$\dfrac{a^4}{b^2}+\dfrac{b^4}{c^2}+\dfrac{c^4}{a^2}\geqslant \sqrt{3\left(a^7+b^7+c^7\right)}$$

Có phải em đưa ra ý tưởng bài này từ bài toán của VasC:

 

$$(a^2+b^2+c^2) \geq \sqrt{3(a^3b+b^3c+c^3a)}$$ :D




#589752 $\sum \frac{a}{\sqrt{a+b}}...

Đã gửi bởi binhnhaukhong on 19-09-2015 - 11:38 trong Bất đẳng thức - Cực trị

Bài toán: Cho $a,b,c$ không âm.Chứng minh rằng:

 

$9(a^3+b^3+c^3)+75(ab^2+bc^2+ca^2) \geq 53(a^2b+b^2c+c^2a)+93abc$

 

Bài toán kiểu này đã quá quen thuộc. Giả sử $a=min${$a,b,c$} và đặt $b=a+x,c=a+y (x,y \geq 0)$ và khai triển ta sẽ được điều luôn đúng.

 

Quay trở lại với bài toán ban đầu, bình phương cả 2 vế BĐT cần chứng minh thì bài toán trở thành:

 

$\sum \frac{a^2}{a+b}+2\sum \frac{ab}{\sqrt{(a+b)(b+c)}}\leq \frac{25}{16}\sum a $

Áp dụng BĐT Cauchy-Schwarz ta có:

$\sum \frac {ab}{\sqrt {(a + b)(b + c)}} \leq \sqrt {\left( \sum ab\right) \left( \sum \frac {ab}{(a + b)(b + c)}\right)} = \sqrt {\frac {\left( \sum ab \right) \left( \sum a^2b + 3abc\right)}{(a + b)(b + c)(c + a)}}$

 

Vậy nên ta sẽ đi chứng minh là:

 

$2\sqrt{\frac{(\sum ab)(\sum a^2b+3abc)}{\prod (a+b)}}\leq \frac{9}{16}\sum a+\sum \frac{ab}{a+b}$

Áp dụng bài toán ở trên thì ta có:

$ \sum a^2b + 3r \leq \frac {3(3 + 16q + 31r)}{128}.$

 

Chuẩn hóa $a+b+c=1$ và chuyển bài toán về dạng $p,q,r$ thì ta cần chứng minh rằng:

 

$2\sqrt {\frac {3q(3 + 16q + 31r)}{128(q - r)}} \le \frac {9}{16} + \frac {q^2 + r}{q - r}$  :) 




#589322 $\sum \frac{a^2+b^2}{(a+bc)(b+ca)}\ge...

Đã gửi bởi binhnhaukhong on 16-09-2015 - 18:11 trong Bất đẳng thức và cực trị

Cách 2: Áp dụng cho cả bài toán làm mạnh.

 

Ta sẽ chứng minh BĐT sau:

 

$\sum \frac{a^2}{(a+bc)(b+ca)}+\sum \frac{b^2}{(a+bc)(b+ca)}\geq \frac{3}{2}$

 

Áp dụng Cauchy-Schwarz thì ta cần chứng minh rằng:

 

$\frac{2(a+b+c)^2}{\sum (a+bc)(b+ca)}\geq \frac{3}{2}$

 

BĐT này tương đương với:

$\left(a+b+c-3 \right)\left [ 4(a+b+c)+3-3abc \right ] \geq 0$

 

Spoiler




#589320 $\sum \frac{a^2+b^2}{(a+bc)(b+ca)}\ge...

Đã gửi bởi binhnhaukhong on 16-09-2015 - 18:04 trong Bất đẳng thức và cực trị

Có 2 cách cho bài này.

 

Cách 1: Ta chứng minh rằng:

 

$\sum {\frac{{{a^2} + {b^2}}}{{\left( {a + bc} \right)\left( {b + ca} \right)}}} \ge \sum {\frac{2}{{{{\left( {1 + c} \right)}^2}}}} $

 

Bất đẳng thức này suy ra từ:

 

$2\left( {a + bc} \right)\left( {b + ca} \right) \le \left( {{a^2} + {b^2}} \right){\left( {1 + c} \right)^2}$

 

Do vậy ta sẽ chứng minh rằng:

$\frac{1}{(a+b)^2}+\frac{1}{(c+b)^2}+\frac{1}{(c+a)^2}\geq \frac{3}{4}$ với $ab+bc+ac=3$

 

Tiếp tục đặt $x=\frac{1}{(a+b)},y=\frac{1}{(c+b)},z=\frac{1}{(a+c)}$

 

Ta chuyển bài toán về chứng minh $x^2+y^2+z^2 \geq \frac{3}{4} (1)$ với $x+y+z=2(xy+yz+xz)$

 

Cộng cả 2 vế BĐT $(1)$ với $2(xy+yz+xz)$ thì ta sẽ chỉ ra $(x+y+z)^2 \geq \frac{3}{4}+(x+y+z)$

 

BĐT này tương đương với: $\left[ {2\left( {x + y + z} \right) - 3} \right]\left[ {2\left( {x + y + z} \right) + 1} \right] \ge 0$ đúng do $x+y+z \geq \frac{3}{2}$




#588592 [BĐT HSGS TST ngày 1 vòng 1] $\sum \frac{1}{(a+...

Đã gửi bởi binhnhaukhong on 12-09-2015 - 20:49 trong Bất đẳng thức - Cực trị

Bài này chính là bài toán của anh Trần Quốc Anh và có vẻ như bài VMO 2014 cũng xây dựng trên ý tưởng bài toán này.




#586519 $\left | ab(a^{2} - b^{2}) + bc(b^{2}...

Đã gửi bởi binhnhaukhong on 31-08-2015 - 22:49 trong Bất đẳng thức và cực trị

Bài viết mang tính chất gợi ý:

 

Ta có thể chuyển bài toán thành tìm GTLN của $P$ trong đó:

 

$P=\frac{\left | (a+b+c)(a-b)(b-c)(c-a)\right |}{(a^2+b^2+c^2)^2}$

 

Bởi tính thuần nhất có thể lấy $p=1$ và áp dụng đẳng thức sau:

 

$\left| (a-b)(b-c)(c-a)\right |=\sqrt{q^2-4q^3+2(9q-2)r-27r^2}$ 

 

Khi đó $P=\frac{\sqrt{q^2-4q^3+2(9q-2)r-27r^2}}{(1-2q)^2}\leq \frac{2(1-3q)\sqrt{3(1-3q)}}{9(1-2q)^2}=f(q)$




#586516 $a,b,c\geqslant0:a^2+b^2+c^2=3.CMR:21+18abc\geqslant 13\s...

Đã gửi bởi binhnhaukhong on 31-08-2015 - 22:34 trong Bất đẳng thức - Cực trị

Dùng BĐT Schur bậc 4:

 

$18abc \geq \frac{3\left[(a+b+c)^2-6\right][(a+b+c)^2+3)]}{2(a+b+c)}$

 

Đến đây chia 2 trường hợp là được. (Ta luôn có $p \geq \sqrt{3}$)

Spoiler




#571677 $\frac{a^2}{b}+\frac{b^2}{c...

Đã gửi bởi binhnhaukhong on 12-07-2015 - 10:19 trong Bất đẳng thức - Cực trị

Bài 1: Cho $x,y,z$ không âm sao cho $xy+yz+xz>0$.Chứng minh rằng:

 

$(xy+yz+xz)\left [ \frac{1}{(x+y)^2}+\frac{1}{(y+z)^2}+\frac{1}{(x+z)^2} \right ]\geq \frac{9}{4}+\frac{xyz(x^3+y^3+z^3-3xyz)}{(x+y)^2(y+z)^2(z+x)^2}$

 

Bài 2: Cho $a,b,c>0$ thỏa $a+b+c=3$.Chứng minh rằng:

 

$\frac{a^2}{b}+\frac{b^2}{c}+\frac{c^2}{a}+4(ab+bc+ac)\geq 15$

 

Bài 3: Cho $a,b,c$ không âm thỏa $a+b+c=3$.Chứng minh rằng:

 

$a^3b+b^3c+c^3a+\frac{1419abc}{256}\leq \frac{2187}{256}$

 

Bài 4: Cho $a,b,c$ thực dương.Chứng minh rằng:

 

$\frac{a^3+b^3+c^3}{8abc}+\frac{a(b+c)}{b^2+c^2}+\frac{b(a+c)}{a^2+c^2}+\frac{c(a+b)}{a^2+b^2}\geq \frac{27}{8}$

 

Bài 5: Cho $a,b,c>0, a^2+b^2+c^2=3$.Chứng minh rằng:

 

$\frac{a^2}{b}+\frac{b^2}{c}+\frac{c^2}{a}+2abc\geq 5$

 

Bài 6: Cho $a,b,c$ thực dương.Chứng minh rằng:

 

$\sum \frac{a^2}{b^2+bc+c^2}\geq \frac{(a^2+b^2+c^2)(a^3+b^3+c^3+3abc)}{(ab+bc+ac)\left [ \sum ab(a+b) \right ]}$

 

Bài 7: Cho $a,b,c>0$ thỏa $a+b+c=1$.Chứng minh rằng:

 

$\sqrt{a^2+abc}+\sqrt{b^2+abc}+\sqrt{c^2+abc}\leq \frac{2}{\sqrt{3}}$

 

Bài 8: $a,b,c\geq 0, a^2+b^2+c^2=4$:

 

$a^3b+b^3c+c^3a\leq \frac{256}{75}+\frac{16}{15}(ab+bc+ac)$

 

Bài 9: $a,b,c\geq 0,a^2+b^2+c^2=1$

 

$\frac{1-ab}{7-3ac}+\frac{1-bc}{7-3ab}+\frac{1-ca}{7-3bc}\geq \frac{1}{3}$

 

Bài 10: $a,b,c$ là các số thực khác 0 .$a^2+b^2+c^2=1$.Chứng minh:

 

$\frac{1-3ab}{1-2ac}+\frac{1-3bc}{1-2ab}+\frac{1-3ca}{1-2bc}\geq 0$

 

Spoiler




#571634 $a^2+b^2+c^2+abc=4$

Đã gửi bởi binhnhaukhong on 12-07-2015 - 09:11 trong Bất đẳng thức và cực trị

 

Bài 4: Cho $x,y,z>0$.Chứng minh rằng:

 

$\frac{xy+yz+xz}{x^2+y^2+z^2}+\frac{(x-y)^2(y-z)^2(z-x)^2}{\prod (x^2-xy+y^2)}\leq 1$

 

Do tính thuần nhất chuẩn hóa $x+y+z=1$. Khi đó BĐT tương đương với:

 

$f(r)=r^2(19-30q)+r(6q^2-7q+1)+q^4\geq 0$

 

Có $r_{ct}=\frac{(1-q)(6q-1)}{2(19-30q)}$

 

Nếu $r_{ct}<0 \Rightarrow q\leq \frac{1}{6}$ thì $f(0)=q^4\geq 0$

 

Nếu $r_{ct} \geq 0 \Leftrightarrow q\geq \frac{1}{6}$ thì $\Delta _{f}=(q+1)(2q-1)(3q-1)(20q^2-10q+1)< 0$

 

Mà $a=19-30q>0$ Vậy nên BĐT đúng.




#571615 $\sum a \sqrt{b+3c^{2}} \leq \sq...

Đã gửi bởi binhnhaukhong on 12-07-2015 - 08:40 trong Bất đẳng thức và cực trị

Ta sẽ chứng minh rằng với cùng điều kiện thì ta có:

 

$2(ab+bc+ac)+5(a^2b+b^2c+c^2a)\leq \frac{11}{9}$

 

Chuyển dang ngôn ngữ $p,q,r$ thì bài toán tương đương:

 

$f(r)=900r^2+(\frac{520}{3}-720q)r+100q^3+56q^2-44q+\frac{484}{81}\geq 0$

 

(Không dùng cách này thì chuyển về đồng bậc xong BW cũng được)

 

Đây là hàm bậc 2 đối với $r$ ta xét 2 trường hợp:

 

$r_{min}=\frac{2100q-520}{6}<0$ thì chứng minh $f(0)\geq 0$

 

$r_{min}=\frac{2100q-520}{6}>0$ thì ta chứng minh $f(r_{min})\geq 0$

 

Trở lại với bài toán ban đầu ta có:

 

$(\sum a\sqrt{b+3c^2})^2\leq (\sum ac+\sum a)(\sum \frac{a(b+3c^2)}{1+c})=(1+ab+bc+ac)(\sum \frac{ab}{1+c}+3\sum ab-3\sum \frac{ac}{1+c})$

 

Lại áp dụng Cauchy-Schwarz lần nữa ta được:

 

$(1+ab+bc+ac)(\sum \frac{ab}{1+c}+3\sum ab- \frac{3(ac+ab+bc)^2}{\sum ab+\sum a^2b})$

 

Lại có: $\sum \frac{ab}{1+c}=ab+bc+ac-abc\sum \frac{1}{c+1}\leq \sum ab-\frac{9abc}{4}$

 

Và từ bài toán phụ ta có $a^2b+b^2c+c^2a\leq \frac{11}{45}-\frac{2}{5}\sum ab$

 

Do đó ta sẽ đi chứng minh:

 

$\frac{2}{3(q+1)}-4q+\frac{9r}{4}+\frac{135q^2}{27q+1}\geq 0$

 

Đến đây có thể dùng Schur để kiểm chứng.




#571574 $a^2+b^2+c^2+abc=4$

Đã gửi bởi binhnhaukhong on 11-07-2015 - 23:41 trong Bất đẳng thức và cực trị

Bài 2 là bài toán mạnh hơn kết quả sau:

 

 

Bài toán: Cho $a,b,c$ không âm thỏa mãn không có 2 số nào đồng thời bằng không.Chứng minh rằng:

 

$\frac{a}{(b+c)^2}+\frac{b}{(a+c)^2}+\frac{c}{(a+b)^2}+\frac{2(ab+bc+ac)}{(a+b)(b+c)(a+c)}\geq \frac{3(a+b+c)}{2(ab+bc+ac)}$

 

Spoiler




#571572 $a^2+b^2+c^2+abc=4$

Đã gửi bởi binhnhaukhong on 11-07-2015 - 23:36 trong Bất đẳng thức và cực trị

 

 

 

 

Bài 3: Cho $a,b,c>0$ và $a^2+b^2+c^2\geq a+b+c$.Chứng minh rằng:

 

$a^3+b^3+c^3+abc\geq ab+bc+ac+1$

 

 

 

 

Đưa BĐT cần chứng minh về dạng đồng bậc là:

 

$a^3+b^3+c^3+3abc\geq [ab+bc+ac+\frac{(a^2+b^2+c^2)^2}{(a+b+c)^2}].\frac{a^2+b^2+c^2}{a+b+c}$

 

Ta có thể bỏ qua điều kiện bài toán ban đầu và chuẩn hóa $a+b+c=1$.

 

BĐT cần chứng minh tương đương với: $2r+4q^3-5q^2+q\geq 0$

 

Xét $q\leq \frac{1}{4}\Rightarrow r\geq 0$ thì BĐT tương đương với: $q(4q-1)(q-1)\geq 0$

 

Xét $q\in \left [ \frac{1}{4} ,\frac{1}{3}\right ]$ thì $r\geq \frac{4q-1}{9}$

 

BĐT cần chứng minh tương đương với: $(3q-2)(3q-1)(4q-1)\geq 0$

 

Tóm lại BĐT cần chứng minh đúng. Dấu bằng khi $a=b=c=1$




#571570 $a^2+b^2+c^2+abc=4$

Đã gửi bởi binhnhaukhong on 11-07-2015 - 23:19 trong Bất đẳng thức và cực trị

Bài 1: Cho $a,b,c$ không âm thỏa $a^2+b^2+c^2+abc=4$.Chứng minh rằng:

 

$i,a^3+b^3+c^3+3(ab+bc+ac)\leq 12$

 

 

Trước hết ta đã biết bài toán quen thuộc:

 

$a^2+b^2+c^2+2abc+1\geq 2(ab+bc+ac)$.

 

Do đó ta dễ dàng có được $a+b+c\leq 3$

 

Và ta có BĐT cần chứng minh tương đương với:

 

$a^3+b^3+c^3-3abc\leq 3(4-abc)-3(ab+bc+ac)=3(a^2+b^2+c^2-ab-bc-ac)$

 

 

Áp dụng hằng đẳng thức thì BĐT này tương đương với $a+b+c\leq 3$




#571566 $a^2+b^2+c^2+abc=4$

Đã gửi bởi binhnhaukhong on 11-07-2015 - 23:11 trong Bất đẳng thức và cực trị

 1-iii.

 Từ giả thiết, đặt 

$$a=\frac{2\sqrt{xy}}{\sqrt{(x+z)(y+z)}};b=\frac{2\sqrt{yz}}{\sqrt{(x+z)(x+y)}};c=\frac{2\sqrt{zx}}{\sqrt{(x+y)(y+z)}}$$

 Thay vào bài toán và ta cần chứng minh :

$$\left [ \sum \frac{4xy}{(x+z)(y+z)} \right ]\left [ \sum \frac{4y\sqrt{xz}}{(x+z)\sqrt{(y+z)(y+x)}} \right ]\leq 9$$

$$\Leftrightarrow 16.\sum xy(x+y).\sum y\sqrt{(xz+yz)(xy+xz)}\leq 9\left [ (x+y)(y+z)(z+x) \right ]^2$$

 Áp dụng BĐT AM-GM ta có :

$$=4\sum y\sqrt{(xz+yz)(xy+xz)}\leq 2\sum (2xyz+y^2z+y^2x)=12xyz+2\sum xy(x+y)$$

Nên ta chỉ cần chứng minh

$$4\left [ 12xyz+2\sum xy(x+y) \right ].\sum xy(x+y)\leq 9\left [ (x+y)(y+z)(z+x) \right ]^2$$

 Áp dụng AM-GM :

$$\left [ 12xyz+2\sum xy(x+y) \right ].4\sum xy(x+y)$$

$$\leq \frac{1}{4}\left [ 6\sum xy(x+y) +12xyz\right ]^2=9\left [ (x+y)(y+z)(z+x) \right ]^2$$

 Kết thúc chứng minh

Chả biết do chữ nó to hay gì mà cách này dài quá em ạ :)

 

Ta đã biết BĐT sau:

 

$abc+2\geq ab+bc+ac$

 

Do đó ta có: $6=2+abc+a^2+b^2+c^2\geq ab+bc+ac+a^2+b^2+c^2\geq 2\sqrt{(a^2+b^2+c^2)(ab+bc+ac)}$.Từ đó ta có đpcm.




#571367 $\frac{2(a^2+b^2+c^2)}{(a+b+c)^2}+\frac...

Đã gửi bởi binhnhaukhong on 11-07-2015 - 10:43 trong Bất đẳng thức - Cực trị

Điểm rơi: $(a,b,c)\sim (1,1,1)$ hoặc $(0,1,1)$ và hoán vị.

Qua thứ một số bất đẳng thức như $a^2b+b^2c+c^2a+abc\leqslant \dfrac{4}{27}(a+b+c)^2$ vân vân và vân vân thì còn cách này, giáp lá càng tới cũng ra :))

Xét $b=\text{median}\{a,c\}$, khi đó điểm rơi nhạy cảm sẽ là $ac=0$ hoặc là $b=c$ hoặc là $a=b$, lúc này: $a^2b+b^2c+c^2a\leqslant b(a^2+ac+c^2)$

Ta cần chứng minh: $\dfrac{2(a^2+b^2+c^2)}{(a+b+c)^2}+\dfrac{ac}{a^2+ca+a^2}\geqslant 1$

Chuẩn hóa $b=1$, đặt $s=a+c, t=ac$ thì bất đẳng thức trở thành: $\dfrac{2s^2-4t+2}{(s+1)^2}+\dfrac{t}{s^2-t}\geqslant 1$

Quy đồng lên ta được: $(s^2-s-2t)^2\geqslant 0$ luôn đúng.

Khỏi đặt ẩn phụ Vieta quy đồng lên chắc là ra $(a^2+c^2-ba-bc)^2\geqslant 0$ theo kết quả đã làm.

Ý tưởng này hay đấy em :). Cách mình thấy tù rồi :).

 

Spoiler




#571363 $\frac{11(a+b+c)}{3}\geq 8\sqrt[3]...

Đã gửi bởi binhnhaukhong on 11-07-2015 - 10:40 trong Bất đẳng thức - Cực trị

pqr hả a

À chuẩn hóa xong dùng định lý $q$ :(